A construction worker pulls a five-meter plank up the side of a building under construction by means of a rope tied to one end of the plank (see figure). Assume the opposite end of the plank follows a path perpendicular to the wall of the building and the worker pulls the rope at a rate of 0.26 meter per second. How fast is the end of the plank sliding along the ground when it is 1.4 meters from the wall of the building? (Round your answer to two decimal places.

Answers

Answer 1

The end of the plank is sliding along the ground at a rate of approximately -0.08 m/s when it is 1.4 meters from the wall of the building. The negative sign indicates that the end of the plank is sliding in the opposite direction.

To find how fast the end of the plank is sliding along the ground, we can use related rates. Let's consider the position of the end of the plank as it moves along the ground.

Let x be the distance between the end of the plank and the wall of the building, and y be the distance between the end of the plank and the ground. We are given that dx/dt = 0.26 m/s, the rate at which the worker pulls the rope.

We can use the Pythagorean theorem to relate x and y:

x² + y² = 5²

Differentiating both sides of the equation with respect to time, we get:

2x(dx/dt) + 2y(dy/dt) = 0

At the given moment when x = 1.4 m, we can substitute this value into the equation above and solve for dy/dt, which represents the rate at which the end of the plank is sliding along the ground.

2(1.4)(0.26) + 2y(dy/dt) = 0

2(0.364) + 2y(dy/dt) = 0

0.728 + 2y(dy/dt) = 0

2y(dy/dt) = -0.728

dy/dt = -0.728 / (2y)

To find y, we can use the Pythagorean theorem:

x² + y² = 5²

(1.4)² + y² = 5²

1.96 + y² = 25

y² = 23.04

y = √23.04 ≈ 4.8 m

Substituting y = 4.8 m into the equation for dy/dt, we have:

dy/dt = -0.728 / (2 * 4.8) ≈ -0.0757 m/s

Know more about distance here:

https://brainly.com/question/13034462

#SPJ11


Related Questions

what correctly displays a realationship between sets of real numbers

Answers

A relationship between sets of real numbers can be accurately represented through mathematical concepts such as subsets, intersections, unions, and equalities.

When comparing sets of real numbers, various mathematical concepts help express the relationship between them. One fundamental concept is the subset. A set A is considered a subset of another set B if every element in A is also an element in B. This relationship is denoted as A ⊆ B. For example, if A = {1, 2} and B = {1, 2, 3}, then A is a subset of B since all the elements in A are also present in B.

Another useful concept is the intersection of sets. The intersection of sets A and B, denoted as A ∩ B, refers to the set of elements that are common to both sets. For instance, if A = {1, 2, 3} and B = {2, 3, 4}, the intersection of A and B would be {2, 3} since those are the elements shared by both sets.

Furthermore, the union of sets provides a way to combine elements from multiple sets. The union of sets A and B, denoted as A ∪ B, represents the set that contains all the elements from both sets without duplication. For example, if A = {1, 2, 3} and B = {3, 4, 5}, the union of A and B would be {1, 2, 3, 4, 5}.

Lastly, the concept of equality between sets implies that two sets have exactly the same elements. If all the elements of set A are present in set B, and vice versa, then A = B. However, it's important to note that the order of elements within a set is irrelevant for equality.

By utilizing these mathematical concepts, one can accurately represent and analyze the relationship between sets of real numbers.

Learn more about subsets here:

https://brainly.com/question/28705656

#SPJ11

Which matrix represents reflection about the ry-plane?

Answers

The matrix that represents reflection about the yz-plane, also known as the ry-plane, is:

[ -1  0  0 ]
[  0  1  0 ]
[  0  0 -1 ]

To understand the matrix that represents reflection about the yz-plane (ry-plane), we need to consider the coordinate system. In a three-dimensional Cartesian coordinate system, the yz-plane is a plane that lies parallel to the x-axis. Reflection about this plane involves flipping the sign of the x-coordinate while leaving the y and z coordinates unchanged.
The matrix representation of this reflection operation can be obtained by considering the effect it has on the standard basis vectors. The standard basis vectors are the vectors that have a single component equal to 1, and all other components equal to 0. In this case, we consider the basis vectors i, j, and k, which represent the unit vectors along the x, y, and z axes, respectively.
When the reflection operation is applied to these basis vectors, the resulting vectors are:i -> -i
j -> j
k -> -k
By arranging these resulting vectors as columns of a matrix, we obtain the reflection matrix for the yz-plane:[ -1  0  0 ]
[  0  1  0 ]
[  0  0 -1 ]
Therefore, this matrix represents reflection about the yz-plane or ry-plane in a three-dimensional Cartesian coordinate system.

learn.more about matrix here

https://brainly.com/question/29132693



#SPJ11

the quadratic formula gives which roots for the equation 2x^2 7x=-2

Answers

The quadratic formula gives the roots -7.17 and 0.17 for the equation [tex]2x^2[/tex] + 7x = -2.

To find the roots of the quadratic equation [tex]2x^2[/tex]+ 7x = -2, we can use the quadratic formula, which states that for an equation of form [tex]ax^2[/tex] + bx + c = 0, the roots can be found using the formula:

x = (-b ± √([tex]b^2[/tex] - 4ac)) / (2a)

In the given equation, we have a = 2, b = 7, and c = -2. Plugging these values into the quadratic formula, we get:

x = (-7 ± √([tex]7^2[/tex] - 4(2)(-2))) / (2(2))

= (-7 ± √(49 + 16)) / 4

= (-7 ± √65) / 4

Calculating the square root of 65, we get √65 ≈ 8.06. Substituting this value back into the equation, we have:

x ≈ (-7 ± 8.06) / 4

This gives us two possible solutions:

x ≈ (-7 + 8.06) / 4 ≈ 1.06 / 4 ≈ 0.27

and

x ≈ (-7 - 8.06) / 4 ≈ -15.06 / 4 ≈ -3.76

Therefore, the roots of equation [tex]2x^2[/tex] + 7x = -2 are approximately x = -3.76 and x = 0.27, rounded to two decimal places.

Learn more about quadratic equation here:

https://brainly.com/question/29269455

#SPJ11

Problem Three. Evaluate
∫∫Ώ (x + y)² dxdy
where isΏ the parallelogram bounded by the lines 2x + 3y = 1, 2x + 3y - 3 3x - 2y = 0, 3x - 2y = 4.

Answers

The parallelogram bounded by the lines 2x + 3y = 1, 2x + 3y - 3 3x - 2y = 0, 3x - 2y = 4,0 ≠ -4, there is no intersection point between these two lines.

The double integral ∫∫Ώ (x + y)² dxdy over the region Ώ, which is the parallelogram bounded by the lines 2x + 3y = 1, 2x + 3y - 3 = 0, 3x - 2y = 0, and 3x - 2y = 4,  to find the limits of integration for x and y.

To determine the limits of integration,  the intersection points of the given lines.

The intersection of the lines 2x + 3y = 1 and 2x + 3y - 3 = 0:

Subtracting the second equation from the first equation,

(2x + 3y) - (2x + 3y - 3) = 1 - 0

3 = 1

Since 3 ≠ 1, there is no intersection point between these two lines.

find the intersection of the lines 2x + 3y = 1 and 3x - 2y = 0:

Solving the system of equations,

2x + 3y = 1 ...(1)

3x - 2y = 0 ...(2)

Multiplying equation (1) by 3 and equation (2) by 2,

6x + 9y = 3 ...(3)

6x - 4y = 0 ...(4)

Subtracting equation (4) from equation (3),

(6x + 9y) - (6x - 4y) = 3 - 0

13y = 3

Simplifying,

y = 3/13

Substituting this value of y into equation (2),  solve for x:

3x - 2(3/13) = 0

3x = 6/13

x = 2/13

Therefore, the intersection point of the lines 2x + 3y = 1 and 3x - 2y = 0 is (x, y) = (2/13, 3/13).

the intersection of the lines 3x - 2y = 0 and 3x - 2y = 4:

Subtracting the second equation from the first equation,

(3x - 2y) - (3x - 2y) = 0 - 4

0 = -4

To know more about parallelogram  here

https://brainly.com/question/28854514

#SPJ4

You measure the weight of 60 randomly chosen backpacks, and find they have a mean weight of 39 ounces. Assume the population standard deviation is 8.9 ounces. Based on this, what is the maximal margin

Answers

Therefore, the maximal margin of error is approximately 2.3 ounces. Hence, the answer is "The maximal margin of error is approximately 2.3 ounces."

We are given: The mean weight of 60 randomly chosen backpacks is 39 ounces.

The population standard deviation is 8.9 ounces. We have to find the maximal margin.

A maximal margin of error represents the maximum distance between the true population parameter and the point estimate, and it is typically expressed as a percentage of the true value.

The formula to calculate the maximal margin of error is given by,

margin of error = Z_α/2* σ/ √n

where Z_α/2 is the critical value for the confidence level α.

To calculate Z_α/2, we use the Z-score table, which shows the percentage of the standard normal distribution that is below a given value of Z.

Since we are not given any confidence level, we assume a 95% confidence level.

For a 95% confidence level, α = 0.05, and the critical value is Z_α/2 = 1.96.

Substituting the values in the formula, we get margin of error = 1.96 * 8.9 / √60= 2.2966.. ≈ 2.3 ounces

To know more about standard deviation visit:

https://brainly.com/question/13336998

#SPJ11

(b) Find the greatest number that divides 300, 560 and 500 without leaving a remainder. ​

Answers

Greatest number that divides 300, 560 and 500 is 20 .

Given numbers : 300, 560 and 500

First let’s find prime factors of 300,560 and 500

300 = 2^2 *3^1 *5^2

560= 2^4 * 7^1 *5^1

500 = 2^2 * 5^3

So,

Here highest common power of 2 is 2

Here highest common power of 3 is 0

Here highest common power of 5 is 1

Here highest common power of 7 is 0

Thus HCF (300, 560 and 500) = 2^2 * 5^1 * 3 ^0 * 7 ^0

=4*5*1*1

= 20

Know more about HCF,

https://brainly.com/question/26431349

#SPJ1

A painter needs to find the area of the gable end of a house. What is the area of the gable if it is a triangle with two sides of 42 ft that meet at a 105° angle?

Answers

The area of the gable end of the house is approximately 868.32 square feet.

To find the area of a triangle, we use the formula:Area = (1/2) x base x Height Where the base is one of the sides of the triangle, and the height is the perpendicular distance from the base to the opposite vertex.

Given that the triangle is the gable end of a house, we assume that the two sides of the triangle are the sides of the roof, and the 105° angle is the angle between the roof and the vertical wall of the house.

Thus, the height of the triangle is the distance between the roof and the wall of the house.Let's draw a diagram to illustrate this:Now we need to find the height of the triangle.

We can do this by using trigonometry, specifically the sine function:

sin 105° = opposite/hypotenuse where the opposite side is the height we want to find, and the hypotenuse is one of the sides of the triangle that we know:

sin 105° = height/42Rearranging,

we get:height = sin 105° x 42

Using a calculator, we find that:height ≈ 40.96 Ft Now we can plug in the values for the base and height into the formula for the area of a triangle:

Area = (1/2) x base x height Area

= (1/2) x 42 x 40.96Area ≈ 868.32 square feet

To learn more about : area

https://brainly.com/question/25292087

#SPJ8

Let f(x) = (x + 1)² Give the largest domain on which f is one-to-one and non-increasing. ___
Give the range of f. ___
Find the inverse of f restricted to the domain above. f-¹(x) = ___
Give the domain of f-¹. ___
Give the range of f-¹. ___

Answers

The function f(x) = (x + 1)² is given, and we need to determine its domain on which it is one-to-one and non-increasing, as well as find its range. Additionally, we need to find the inverse of f restricted to its domain, determine the domain of the inverse function, and find its range.

To find the domain on which f is one-to-one and non-increasing, we need to consider the behavior of the function. The function f(x) = (x + 1)² is a quadratic function with a vertex at (-1, 0) and opens upward. Since it is a one-to-one function, it means that it passes the horizontal line test, and each y-value corresponds to a unique x-value. Therefore, the largest domain on which f is one-to-one and non-increasing is the set of all real numbers, (-∞, ∞).

Next, let's find the range of f. Since the function is a quadratic that opens upward, its minimum value occurs at the vertex (-1, 0), and it increases as x moves away from the vertex. Hence, the range of f is [0, ∞), including zero and all positive real numbers. To find the inverse of f restricted to its domain, we interchange the roles of x and y in equation f(x) = (x + 1)² and solve for y. Let's proceed with the steps:

y = (x + 1)²

Swap x and y:

x = (y + 1)²

Take the square root of both sides:

√x = y + 1

Subtract 1 from both sides:

√x - 1 = y

Therefore, the inverse function of f, restricted to its domain (-∞, ∞), is given by f⁻¹(x) = √x - 1. The domain of f⁻¹ is the set of all non-negative real numbers, [0, ∞) since we took the square root, which requires non-negative values. Lastly, the range of f⁻¹ is the set of all real numbers, (-∞, ∞), because as x varies from 0 to ∞, the square root of x produces values from 0 to ∞, and subtracting 1 does not restrict the range. In summary, the largest domain on which f is one-to-one and non-increasing is (-∞, ∞), the range of f is [0, ∞), the inverse function f⁻¹(x) = √x - 1 has a domain of [0, ∞), and its range is (-∞, ∞).

Learn more about square root here:- brainly.com/question/29286039

#SPJ11








Find the flux of the curl of field F through the shell S. F4yi + 3zj-9xk; S: r(r, 0) = r cos 0i+r sin 0j + (36-r2)k, 0s r s 6 and 0 ≤ 0 ≤ 2π

Answers

The flux of the curl of field F through the given shell S is zero. This means that the net flow of the curl through the shell is negligible.

To find the flux of the curl of field F through the shell S, we need to evaluate the surface integral of the dot product between the curl of F and the outward unit normal vector of the shell S. The curl of F is given as (4y)i + (3z)j - (9x)k.

The shell S is defined by the vector function r(r, θ) = r cos θi + r sin θj + (36 - r^2)k, where r varies from 0 to 6 and θ varies from 0 to 2π. This describes a hollow cylindrical surface centered at the origin with radius 6 and height 72.

The outward unit normal vector to the shell S can be determined using the cross product of the partial derivatives of r with respect to r and θ. By calculating the cross product and normalizing the resulting vector, we obtain the outward unit normal vector n.

Now, we can compute the curl of F, which is (4y)i + (3z)j - (9x)k. Taking the dot product of the curl with the outward unit normal vector n and integrating over the surface S, we find that the flux of the curl through the shell is zero. This indicates that the net flow of the curl through the shell is balanced, resulting in no net flux.

To learn more about flux click here: brainly.com/question/14527109

#SPJ11

Determine the vector and parametric equations of the line going through the points P(1,2,4) and Q(1,3,6). Question 17 (3 points) Do the lines L1​:r=(1,7,−5)+s(2,−2,5),s∈R, and the line L2​:r=(−2,3,−6)+s(3,2,6),s∈R, determine a plane?

Answers

The equation of the plane is:r = (1, 7, −5) + s(2, −2, 5) + t(3, 2, 6)

Where s, t ∈ R.

Solution: The vector and parametric equations of the line going through the points P(1, 2, 4) and Q(1, 3, 6) are given below: Vector Equation :We will determine the direction vector by subtracting the coordinates of two points Q and P.

r = OP + t PQ= (1, 2, 4) + t (0, 1, 2)

Here, OP is the position vector of P, and PQ is the vector from P to Q.

The direction vector of the line L is PQ (0, 1, 2).Parametric Equation:

Now we will express the vector equation in parametric form.

x = 1 + 0ty = 2 + t, and z = 4 + 2

t where t ∈ R.  the lines L1​: r = (1, 7, −5) + s(2, −2, 5), s ∈ R, and

the line L2​: r = (−2, 3, −6) + s(3, 2, 6), s ∈ R, determine a plane.

Let us find two points that lie on both of these lines to find the plane of intersection:

Let point A lie on line L1, such that A = (1, 7, −5)Let point B lie on line L2, such that B = (−2, 3, −6)

Equation of line L1 is given as:r1 = (1, 7, −5) + s(2, −2, 5)

Let's find two values of s such that r1 lies on line L2:r1 = (1, 7, −5) + s(2, −2, 5)= (1 + 2s, 7 − 2s, −5 + 5s)

Now we can equate the two vectors r1 and r2:r1 = r2⟹(1 + 2s, 7 − 2s, −5 + 5s) = (−2 + 3t, 3 + 2t, −6 + 6t)From this system of equations,

we can determine the values of s and t such that the two points coincide and lie on both lines.

Now we solve the system of equations:1 + 2s = −2 + 3t7 − 2s = 3 + 2t−5 + 5s = −6 + 6tSolving the system,

we get: s = −1 and t = 1

We can check if the points A and B lie on both lines:L1, s = −1: r1 = (−1, 9, 0)L2, t = 1: r2 = (1, 5, 0)

We can see that the two points A and B both lie on the plane with the equation: r = r0 + s v1 + t v2

Where r0 is the position vector of A, and v1, v2 are the direction vectors of the lines L1 and L2, respectively.

Substituting the values:r0 = (1, 7, −5)v1 = (2, −2, 5)v2

= (3, 2, 6)

To know more about equation visit:

https://brainly.com/question/29538993

#SPJ11








Problem 3. Find the mass and center of mass of the lamina that occupies the region bounded by the parabolas y = r² and x = y², and has density function p(x, y) = √√T.

Answers

The mass and center of mass of the lamina that occupies the region D and has the given density function p is 57/14 and (14/27, 7/18) respectively.

The center of mass (x―,y―) of a lamina with density function ρ(x,y) is given by

x = M(y)/m, y = M(x)/m

Where, m=∫∫ρ(x,y)dA

Mx=∫∫ yρ(x,y)dA

My=∫∫ xρ(x,y)dA

Given that, D is bounded by y=x^2 and x=y^2

And ρ(x,y)=19√x

Now, for the point of intersection of y=x^2,x=y^2

The lamina is customary, so its focal point of mass is its mathematical focus. Take a lamina with three holes near its perimeter and now suspend it through each hole one at a time.

Here,

The mass density of a lamina is the mass per unit area. Take into consideration the following lamina, whose density varies with the object: On a semicircular lamina D with a radius of three, the density at any point is proportional to the distance from the origin.

We know,

A lamina's centroid is the point at which it would balance on a needle. The point at which a solid would "balance" is called the centroid.

Consider a lamina formed by the intersection of two curves y = f (x) and y = g (x) at points with x-coordinates of x = a and b.

Mass (M) = b a g (x) f (x) d y d x x-coordinate (x) = b a x (x, y) [ g (x) f (x)] d x y-coordinate (y) = b a 1 2 (x, y) [ [ g (x)] 2 [ f (x)] 2] d x.

To learn more about lamina's centroid here:

brainly.com/question/20566103

#SPJ1

If x, y, z be in HP prove that (y+x)/(y-x)+(y+z)/(y-z) = 2 ​

Answers

If x, y, and z be in Harmonic progression, then the equation (y+x)/(y-x)+(y+z)/(y-z) = 2 ​is satisfied.

The reciprocal of Harmonic progression (HP) is arithmetic progression (AP),

Let d be a common difference,

1/x, 1/y, and 1/z are in AP.

1/y - 1/x = d

1/z - 1/y = d

where d is the common difference,

Evaluating equations.

(y+x)/(y-x) + (y+z)/(y-z)

[(y+x)(y-z) + (y+z)(y-x)] / [(y-x)(y-z)]

[2y² - 2xz] / [(y-x)(y-z)]

Substituting value of d,

[2y² - 2xz] / [(-d)(d)]

[2y² - 2xz] / (d²) = 2

By solving, we get

y² - xz = d²

The common difference in the AP is equal to the difference between two successive terms.

Therefore, d² = xz and d² = y²

y² - xz = xz

y² = 2xz

= 2

Hence, (y+x)/(y-x)+(y+z)/(y-z) = 2.

To learn more about Harmonic progression, here:

https://brainly.com/question/27927304

#SPJ1

In (r, q) coordinates A = (83.0, 344 degrees) and B = (69.0, 2.90E2 degrees). Given R = = A - B A - (a) In polar coordinates the resultant vector is R = (₁ Rr, Re Ro). What is the radial component, Rr?

Answers

To find the radial component, Rr, of the resultant vector R in polar coordinates, we need to subtract the radial components of the vectors A and B. Rr represents the magnitude of the radial displacement in the polar coordinate system.

In polar coordinates, a vector is represented by its radial distance from the origin (Rr) and its angle from the positive x-axis (Re). We are given the coordinates of vectors A and B in (r, q) form.

Vector A is given as A = (83.0, 344 degrees) and vector B is given as B = (69.0, 290 degrees).

To find the resultant vector R = A - B, we subtract the radial components and add the angular components.

Rr = |RrA - RrB|

= |83.0 - 69.0|

= |14.0|

= 14.0

The radial component, Rr, of the resultant vector R is 14.0 in the given polar coordinate system. It represents the magnitude of the radial displacement or distance from the origin.

Learn more about Vector here:

https://brainly.com/question/24256726

#SPJ11

The following data show the monthly salaries of a sample of IBM
graduates.
IBM Student
Monthly Salary (in 1,000s Rupees)
A. 78
B. 87
C. 80
D. 100
E. 104
F. 88

Answers

The median monthly salary of the IBM graduates in the given data set is 87.5 thousand rupees.

To find the median of the given data set, the first step is to arrange the given data set in ascending order.

The data set is:{78, 87, 80, 100, 104, 88}

After arranging the data set in ascending order, it becomes:{78, 80, 87, 88, 100, 104}

There are six data points in the given data set.

To find the median, the middle data point must be found. In this case, there are two middle data points because there are an even number of data points.

To find the median of the data set, the two middle data points must be averaged.

The two middle data points are 87 and 88.

To find the average of these two data points, add them together and divide by 2:

(87 + 88)/2 = 175/2 = 87.5

Therefore, the median monthly salary of the IBM graduates in the given data set is 87.5 thousand rupees.

Know more about median here:

https://brainly.com/question/26177250

#SPJ11

Customers arrive at the CVS Pharmacy drive-thru at an average rate of 5 per hour. What is the probability that less than 4 customers will arrive at the drive-thru during a randomly chosen hour? 0.600

Answers

The probability that less than 4 customers will arrive at the CVS Pharmacy drive-thru during a randomly chosen hour is 0.600.

Given, the average rate of arrival of customers at the CVS Pharmacy drive-thru = λ = 5 per hour. We need to find the probability of less than 4 customers arriving in a randomly chosen hour.Using Poisson's probability distribution formula,P (x < 4) = e⁻ᵩ [ 1/0! + ᵩ/1! + ᵩ²/2! + ᵩ³/3!]where ᵩ is the expected number of customers arriving during a randomly chosen hour,= 5 since the average rate of arrival of customers at the CVS Pharmacy drive-thru = 5 per hour= e⁻⁵ [1/0! + 5/1! + 5²/2! + 5³/3!] = e⁻⁵ [ 1 + 5 + 12.5 + 20.83]= e⁻⁵ × 39.33= 0.674Thus, the probability that less than 4 customers will arrive at the CVS Pharmacy drive-thru during a randomly chosen hour is 0.674.

The given value of the average rate of arrival of customers at the CVS Pharmacy drive-thru = λ = 5 per hour

Therefore, the expected number of customers arriving during a randomly chosen hour = ᵩ = 5.

Using Poisson's probability distribution formula,P (x < 4) = e⁻ᵩ [ 1/0! + ᵩ/1! + ᵩ²/2! + ᵩ³/3!]P (x < 4) = e⁻⁵ [ 1 + 5 + 12.5 + 20.83]P (x < 4) = e⁻⁵ × 39.33= 0.674

Therefore, the probability that less than 4 customers will arrive at the CVS Pharmacy drive-thru during a randomly chosen hour is 0.674.

To know more about Poisson's probability distribution  visit :-

https://brainly.com/question/32764689

#SPJ11


from Coding theory
n-1 Let q ≥ 2 and n ≥ 2 be any integers. Show that Aq(n, 2) = qª

Answers

In coding theory, it is proven that Aq(n, 2) = qª, where q is greater than or equal to 2 and n is greater than or equal to 2.

To show that Aq(n, 2) = qª, we consider the scenario where q ≥ 2 and n ≥ 2 are integers. The value Aq(n, 2) represents the maximum number of codewords of length n over an alphabet of size q, with a minimum distance of 2.

In this case, to construct a codeword of length n, we have q choices for each position, resulting in q × q × ... × q (n times), which is equal to q raised to the power of n, denoted as qª.

Furthermore, with a minimum distance of 2, any two distinct codewords must differ in at least two positions. Therefore, the maximum number of codewords is qª.

Hence, we have shown that Aq(n, 2) = qª for any integers q ≥ 2 and n ≥ 2, according to the coding theory result.

Learn more about Coding theory here: brainly.com/question/32339938

#SPJ11

suppose that $f(x)$ is a polynomial that has degree $6$ and $g(x)$ is a polynomial that has degree $3$. if $h(x)$ is also a polynomial such that $f(g(x)) g(h(x)) h(f(x))$ is a polynomial of degree $36$, then what is the degree of the polynomial $h$?

Answers

The resulting polynomial will have a degree of is [tex]$g(h(x))$[/tex]a polynomial that results from substituting [tex]$h(x)$ into $g(x)$.[/tex][tex]$(\text{degree of } h(x)) \times 6$.[/tex]

To determine the degree of the polynomial $h(x)$, we need to analyze the degree of the composite polynomial [tex]$f(g(x))g(h(x))h(f(x))$.[/tex]

Let's break down the composite polynomial:

$f(g(x))$ is a polynomial that results from substituting $g(x)$ into $f(x)$. Since $g(x)$ is a polynomial of degree $3$ when substituted into $f(x)$ of degree $6$, the resulting polynomial will have a degree of [tex]$6 \times 3 = 18$.[/tex]

$g(h(x))$ is a polynomial that results from substituting $h(x)$ into $g(x)$. Since $h(x)$ is a polynomial of unknown degree when substituted into $g(x)$ of degree $3$, the resulting polynomial will have a degree of [tex]$3 \times (\text{degree of } h(x))$.[/tex]

$h(f(x))$ is a polynomial that results from substituting $f(x)$ into $h(x)$. Since $f(x)$ is a polynomial of degree $6$ when substituted into $h(x)$ of unknown degree, The resulting polynomial will have a degree of

[tex]$(\text{degree of } h(x)) \times 6$.[/tex]

To know more about degree of the polynomial:- brainly.com/question/31437595

#SPJ11








Find the derivative of the function f(x) = 3x² + 4x - 3 at number 14. (Use symbolic notation and fractions where needed.) f'(14) =

Answers

The given function is;f(x) = 3x² + 4x - 3To find f'(14), we first find the derivative of the given function f(x).We can use the power rule of differentiation to find the derivative of f(x).

According to the power rule, if the function is of the form f(x) = x^n, then the derivative of the function is given by f'(x) = nx^(n-1).So, applying the power rule of differentiation to the given function, we get;f'(x) = 6x + 4Now, we need to find the value of f'(14).So,f'(14) = 6(14) + 4= 88 Therefore, f'(14) = 88.

We get the value of the derivative of the given function at number 14 as 88. The solution can be presented within 150 words as follows:To find the derivative of the function f(x) = 3x² + 4x - 3 at number 14, we first find the derivative of the given function f(x).

We can use the power rule of differentiation to find the derivative of f(x). According to the power rule, if the function is of the form f(x) = x^n, then the derivative of the function is given by f'(x) = nx^(n-1). So, applying the power rule of differentiation to the given function, we get; f'(x) = 6x + 4. Now, we need to find the value of f'(14).

Therefore, f'(14) = 6(14) + 4 = 88. Hence, we get the value of the derivative of the given function at number 14 as 88.

To know more about power rule of differentiation visit:

https://brainly.com/question/30117847

#SPJ11

Determine whether the given function is exponential or not. If it is exponential, identify the value of the base a.
x H(x)
-1 8
0 13
1 18
2 23
3 28
a) exponential a = 13
b) exponential a = 5
c) exponential a = 8
d) not exponential

Answers



the correct answer is (d) not exponential. None of the options (a), (b), or (c) are applicable as they indicate an exponential function with a specific base value, but the given function does not exhibit exponential behavior.

ToTo determine whether the given function is exponential or not, we need to check if there is a consistent pattern in the relationship between x and H(x). Let's calculate the differences between consecutive values of H(x):

ΔH(x) = 13 - 8 = 5
ΔH(x) = 18 - 13 = 5
ΔH(x) = 23 - 18 = 5
ΔH(x) = 28 - 23 = 5

The differences between consecutive values of H(x) are constant, which suggests that the function is linear rather than exponential. Therefore, the correct answer is (d) not exponential. None of the options (a), (b), or (c) are applicable as they indicate an exponential function with a specific base value, but the given function does not exhibit exponential behavior.

 To Learn more about exponential form click here: brainly.com/question/28596571

#SPJ11


Doppler redshift is the redshifting of spectra from objects
moving away from us, and cosmological redshift is the redshifting
of objects moving toward us.
Group of answer choices
True
Ques

Answers

The statement that Doppler redshift is the redshifting of spectra from objects moving away from us, and cosmological redshift is the redshifting of objects moving toward us is false.

Doppler redshift and cosmological redshift are two distinct phenomena related to the observed shift in the wavelength of light emitted by celestial objects. Doppler redshift occurs due to the relative motion between the source of light and the observer. When an object is moving away from the observer, the wavelength of the light it emits appears stretched, resulting in a redshift. Conversely, if the object is moving towards the observer, the wavelength appears compressed, leading to a blueshift.

On the other hand, cosmological redshift is caused by the expansion of the universe. As space itself expands, the wavelengths of light traveling through space also stretch, resulting in a redshift. This redshift is not directly related to the motion of objects towards or away from the observer.

Therefore, the statement that Doppler redshift is associated with objects moving away from us, and cosmological redshift is associated with objects moving towards us is incorrect.

Learn more about redshift here : brainly.com/question/30257423

#SPJ11

A random sample of 100 bottles of water were collected. From the sample, the mean ounces was calculated to be 16.91. In addition, the sample standard deviation was calculated to be 0.12. Which of the following is true about these values?

a. The mean of 16.91 is a parameter and the standard deviation of 0.12 is a statistic.

b. The mean of 16.91 and the standard deviation of 0.12 are statistics.

c. The mean of 16.91 and the standard deviation of 0.12 are parameters.

d. The mean of 16.91 is a statistic and the standard deviation of 0.12 is a parameter.

Answers

c. The mean of 16.91 and the standard deviation of 0.12 are parameters.

In statistics, parameters are values that describe a population. In this case, the mean ounces and the standard deviation calculated from the sample are used to estimate the corresponding parameters of the population. Since the sample was taken from a larger population of bottles of water, the mean of 16.91 ounces and the standard deviation of 0.12 ounces are estimates of the true population parameters. Therefore, they are considered parameters rather than statistics.

A statistic, on the other hand, is a value calculated from a sample and is used to describe or estimate a population parameter. In this scenario, the values calculated from the sample (mean and standard deviation) are used as estimates of the population parameters, making them parameters rather than statistics.

know more about standard deviation here:

https://brainly.com/question/29115611

#SPJ11

Minimize f = x² + 2x2 + 3x3 subject t +3x3 subject to the constraints
8₁=x₁-x₂2x₂ ≤ 12
8₂=x₁ + 2x₂-3x3 ≤8
using Kuhn-Tucker conditions.

Answers

This critical point is a minimum point of the given function subject to the given constraints.

Kuhn-Tucker conditions are the first-order necessary conditions for constrained optimization problems.

To minimize the given function `f(x) = x² + 2x² + 3x³` subject to the constraints `x₁ - x₂²x₂ ≤ 12` and `x₁ + 2x₂ - 3x³ ≤ 8`, we can use the following Kuhn-Tucker conditions:

First-order conditions:∂L/∂x₁ + λ₁∂g₁/∂x₁ + λ₂∂g₂/∂x₁ = 0∂L/∂x₂ + λ₁∂g₁/∂x₂ + λ₂∂g₂/∂x₂ = 0∂L/∂x₃ + λ₁∂g₁/∂x₃ + λ₂∂g₂/∂x₃ = 0∂L/∂λ₁g₁ = 0∂L/∂λ₂g₂ = 0

Here, L(x, λ₁, λ₂) = f(x) + λ₁(g₁(x) - 12) + λ₂(g₂(x) - 8)

Let's first find the partial derivatives of the objective function: ∂f/∂x₁ = 0∂f/∂x₂ = 4x₂∂f/∂x₃ = 9x²

Now, let's find the partial derivatives of the constraint functions:∂g₁/∂x₁ = 1∂g₁/∂x₂ = -2x₂∂g₁/∂x₃ = 0∂g₂/∂x₁ = 1∂g₂/∂x₂ = 2∂g₂/∂x₃ = -3

Using the above expressions, we can write the Kuhn-Tucker conditions as:

1) ∂L/∂x₁ + λ₁(1) + λ₂(1) = 0 ⇒ 0 + λ₁ + λ₂ = 0 ...(i)

2) ∂L/∂x₂ + λ₁(-2x₂) + λ₂(2) = 0 ⇒ 4x₂ - 2λ₁ + 2λ₂ = 0 ...(ii)

3) ∂L/∂x₃ + λ₁(0) + λ₂(-3) = 0 ⇒ 9x² - 3λ₂ = 0 ...(iii)

4) ∂L/∂g₁ = λ₁ = 0 ...(iv)5) ∂L/∂g₂ = λ₂ = 0 ...(v)

From equations (iv) and (v), we get: λ₁ = 0 and λ₂ = 0

Putting these values in equations (i) and (ii), we get: λ₁ + λ₂ = 0 and 2x₂ = λ₁ - λ₂Since λ₁ = λ₂ = 0, we get x₂ = 0From equation (iii), we get 9x² = 0 ⇒ x = 0

Thus, the critical point of the given function subject to the given constraints is x = (0, 0, 0)Now, let's check the second-order condition for this point:∂²L/∂x² = [0 0 0; 0 4 0; 0 0 18] > 0

Hence, this critical point is a minimum point of the given function subject to the given constraints.

Know more about critical point here:

https://brainly.com/question/30459381

#SPJ11

Exercise. For now we focus on the 2nd integral -5x + 1 1₂ dx x² + 4x + 9 It may not be obvious how to proceed. Since the denominator is a quadratic, one path forward is to try trig substitution. First we complete the square on the denominator. x² + 4x +9= x² + 4x +4-4+9= (x+2 )² + 5 I Hence we should use the trig substitution x + 2 = sqrt(5)tan(theta Thus dx = (5)sec^2(theta) do. The integral in terms of 0 is: -5x + 1 S dx ? do = x² + 4x +9 and evaluating this integral, we find: -5√5 tan(0) + 11 √5 do = ? (Leave the result in terms of 0 and use C for the constant of integration)

Answers

The integral of -5x + 1 / (x^2 + 4x + 9) can be evaluated as follows:

Complete the square on the denominator: x^2 + 4x + 9 = (x + 2)^2 + 5.

Substitute x + 2 = sqrt(5) * tan(theta) and dx = sqrt(5) * sec^2(theta) d(theta) in the integral.

The integral becomes -5 * sqrt(5) * tan(theta) + 11 * sqrt(5) / 5.

Integrate this expression with respect to theta to find the antiderivative.

Substitute back theta = tan^(-1)((x + 2) / sqrt(5)) and simplify to express the answer in terms of x.

Unfortunately, without the specific limits of integration or the result of the integration, I cannot provide the final answer.

To know more about Expression visit-

brainly.com/question/14083225

#SPJ11

Solve the following system by the method of reduction.
2x - 6z = 24
x - 3y - 3z = 30
x + y -3z = 6
3x + y + z = 0
Select the correct choice below and, if necessary, fill in the answer boxes to complete your choice.
A. x = ___, y = ___, z = ___
(Type integers or fractions.)
B. x=r, y= ___, z= ___
(Type integers or fractions.)
C. There is no solution.

Answers

In order to solve the given system of equations by the method of reduction, we have to use the following steps:Step 1: Convert the given system of equations into an augmented matrix form.Step 2: Apply the row operations to the augmented matrix to obtain a matrix in the row echelon form.Step 3: Find the solution of the system of equations.The augmented matrix form of the given system of equations is:\[\begin{bmatrix} 2 & 0 & -6 & 24 \\ 1 & -3 & -3 & 30 \\ 1 & 1 & -3 & 6 \\ 3 & 1 & 1 & 0 \end{bmatrix}\]Performing the row operation - R1 + (1/2) R2, we get,\[\begin{bmatrix} 2 & -3/2 & -9/2 & 39 \\ 1 & -3 & -3 & 30 \\ 1 & 1 & -3 & 6 \\ 3 & 1 & 1 & 0 \end{bmatrix}\].

Performing the row operation - R1 + (1/2) R3, we get,\[\begin{bmatrix} 2 & -3/2 & -9/2 & 39 \\ 1 & -3 & -3 & 30 \\ 0 & 5/2 & -9/2 & -33/2 \\ 3 & 1 & 1 & 0 \end{bmatrix}\]Performing the row operation - R1 + (3/2) R4, we get,\[\begin{bmatrix} 2 & -3/2 & -9/2 & 39 \\ 1 & -3 & -3 & 30 \\ 0 & 5/2 & -9/2 & -33/2 \\ 0 & 5 & 11 & -117 \end{bmatrix}\]Performing the row operation - R2 + (1/2) R3, we get,\[\begin{bmatrix} 2 & -3/2 & -9/2 & 39 \\ 1 & -2 & -6 & -3 \\ 0 & 5/2 & -9/2 & -33/2 \\ 0 & 5 & 11 & -117 \end{bmatrix}\]Performing the row operation - (2/5) R3 + R4, we get,\[\begin{bmatrix} 2 & -3/2 & -9/2 & 39 \\ 1 & -2 & -6 & -3 \\ 0 & 5/2 & -9/2 & -33/2 \\ 0 & 0 & 1 & -18 \end{bmatrix}\].

To know more about reduction visit :-

https://brainly.com/question/8963217

#SPJ11

"Set up, but do not evaluate, an integral for the volume of the solid obtained by rotating the region bounded by y = 0, y = sin(x), and 0 < x < π about the line y = -2. Please also provide a sketch of the region and the line of rotation."

Answers

The integral for the volume generated is V = ∫[0, π] 2π(x + 2) [sin(x)] dx

How to set up the integral for the volume generated

From the question, we have the following parameters that can be used in our computation:

y = 0 and y = sin(x)

Also, we have

The line u = -2

Set the equations to each other

So, we have

sin(x) = 0

When evaluated, we have

x = 0 and x = π

For the volume generated from the rotation around the region bounded by the curves, we have

V = ∫[a, b] 2π(x + 2) [g(x) - f(x)] dx

This gives

V = ∫[0, π] 2π(x + 2) [sin(x) - 0] dx

So, we have

V = ∫[0, π] 2π(x + 2) [sin(x)] dx

Hence, the integral for the volume generated is V = ∫[0, π] 2π(x + 2) [sin(x)] dx

Read more about volume at

brainly.com/question/11942113

#SPJ1

This problem illustrates what happens to an unbiased estinator when it undergoes a nonlinear transformation. In Example 2.1, if we choose to estimate the unknown parameter 0 = A² by (Σετ) 9 can we say that the estimator is unbiased? What happens as N ?

Answers

In Example 2.1, the estimator (Σε/N)² for estimating A² is unbiased, as its expected value equals the true parameter value. It remains unbiased as the sample size N increases.



In Example 2.1, we are considering estimating the unknown parameter θ = A² using the estimator (Σε/N)², where ε represents the random error and N is the sample size. To determine if the estimator is unbiased, we need to check if its expected value equals the true parameter value.

The estimator can be rewritten as [(Σε)²]/N². Since the errors ε are assumed to be unbiased with zero mean, E(ε) = 0. Therefore, E(Σε) = N * E(ε) = 0, and the expected value of the estimator becomes E([(Σε)²]/N²) = E(0) = 0.

Thus, we can conclude that the estimator (Σε/N)² is unbiased for estimating A² since its expected value equals the true parameter value.

As the sample size N increases, the sum of errors Σε tends to increase in magnitude, resulting in a larger numerator. However, the denominator N² also increases, which compensates for the increase in the numerator, keeping the estimator unbiased. In other words, the bias of the estimator remains zero even as N increases.

It is worth noting that the consistency of the estimator, i.e., whether it converges to the true value as N approaches infinity, is a separate property that should be examined separately.

To learn more about parameter value click here

brainly.com/question/14283309

#SPJ11

There are 6 different types of drinks in a store and John wants to buy 5 drinks. Find the number of choices John can do this. a) 252 b) 720 c) 6 d) 120 e) 30

Answers

The correct answer is c) 6. John has 6 choices to buy 5 drinks out of the 6 available types.

To find the number of choices John can make when buying 5 drinks out of 6 different types, we can use the concept of combinations. Since the order of drinks doesn't matter, we need to find the number of combinations of 6 drinks taken 5 at a time.

The formula for combinations is given by nCr = n! / (r!(n-r)!), where n is the total number of options and r is the number of choices.

Using this formula, we can calculate the number of choices as 6C5 = 6! / (5!(6-5)!) = 6.

Therefore, the correct answer is c) 6. John has 6 choices to buy 5 drinks out of the 6 available types.

For more information on combinations visit: brainly.com/question/10335643

#SPJ11

Representing a large auto dealer, a buyer attends car auctions. To help with the bidding, the buyer built a regression equation to predict the resale value of cars purchased at the auction. The equation is given below. Estimated Resale Price ($) = 20,000 - 2,050 Age (year), with p = 0.52 and se = $3,200 = Use this information to complete parts (a) through (c) below. (a) Which is more predictable: the resale value of one six-year-old car, or the average resale value of a collection of 16 cars, all of which are six years old? A. The resale value of one six-year-old car is more predictable because only one car will contribute to the error. B. The average of the 16 cars is more predictable by default because it is impossible to predict the value of a single observation. C. The average of the 16 cars is more predictable because the averages have less variation. D. The resale value of one six-year-old car is more predictable because a single observation has no variation. (b) According the buyer's equation, what is the estimated resale value of a six-year-old car? The average resale value of a collection of 16 cars, each six years old? The estimated resale value of a six-year-old car is $ (Type an integer or a decimal. Do not round.) The average resale value of a collection of 16 cars, each six years old is $ (Type an integer or a decimal. Do not round.) (c) Could the prediction from this equation overestimate or underestimate the resale price of a car by more than $2,250? O A. No. Since $2,250 is less than the standard error of $3,200, it is impossible for the regression equation to be off by more than $2,250. B. No. Since $2,250 is greater than the absolute value of the predicted slope, $2,050, it is impossible for the regression equation to be off by more than $2,250. C. Yes. Since $2,250 is less than the standard error of $3,200, it is quite possible that the regression equation will be off by more than $2,250. D. Yes. Since $2,250 is greater than the absolute value of the predicted slope, $2,050, it is quite possible that the regression equation will be off by more than $2,250.

Answers

The estimated resale value of a six-year-old car is $12,200. The prediction from this equation could potentially overestimate or underestimate the resale price of a car by more than $2,250.

(a) The average resale value of a collection of 16 six-year-old cars is more predictable than the resale value of one individual six-year-old car. This is because the average of multiple observations tends to have less variation and is more representative of the overall trend. When taking an average, the individual variations tend to cancel out, resulting in a more reliable estimate.

(b) According to the buyer's equation, the estimated resale value of a six-year-old car is $12,200. The average resale value of a collection of 16 six-year-old cars would be the same, $12,200, since the equation gives a fixed value for each six-year-old car.

(c) Yes, the prediction from this equation could potentially overestimate or underestimate the resale price of a car by more than $2,250. The standard error of the estimate (se) is $3,200, which indicates the typical amount of variation in the predicted values. Since $2,250 is less than the standard error, it is possible for the regression equation to be off by more than $2,250. The absolute value of the predicted slope ($2,050) is not directly related to the potential overestimation or underestimation. The standard error provides a more appropriate measure of the potential variability in the predictions.

Learn more about equation here:

https://brainly.com/question/28243079

#SPJ11

Determine whether the equation represents exponential growth, exponential decay, or neither.

Explain. y = 900(1 - 0)5

O Exponential growth; because the base that is the rate of proportion is greater than 1.

O Exponential growth; because the base that is the rate of proportion is less than 1.

O Exponential decay; because the base that is the rate of proportion is greater than 1.

O Exponential decay; because the base that is the rate of proportion is less than 1.

O Neither; because the equation is not an exponential function.

Answers

The correct answer is: O Neither; because the equation is not an exponential function.

The equation y = 900(1 - 0)5 can be simplified to y = 900(1)5 = 900.

In this case, the equation represents neither exponential growth nor exponential decay. It simply states that the value of y is constant and equal to 900. There is no change or growth/decay occurring over time or any other independent variable.

Know more about exponential function here:

https://brainly.com/question/29287497

#SPJ11

what is the equation of a line that passes through the point (2, −10) and is parallel to 14x 2y=6?

Answers

The equation of line that passes through the point (2, -10) and is parallel to 14x - 2y = 6 is y = -3.5x - 3.

A line parallel to 14x - 2y = 6 will have the same slope as the given line, which can be found by rearranging the equation into slope-intercept form:

14x - 2y = 6-2y = -14x + 6y = 7x - 3y = -3.5x + 1.5

The slope of this line is -3.5,

so the slope of any parallel line will also be -3.5.

We also know that this line passes through the point (2, -10).

Using point-slope form, the equation of the line is:y - y1 = m(x - x1), where m is the slope and (x1, y1) is the given point.

y - (-10) = -3.5(x - 2)y + 10 = -3.5x + 7y = -3.5x - 3

Let's verify that this equation represents a line parallel to the given line:

14x - 2y = 6-2y = -14x + 6y = 7x - 3y = -3.5x + 1.5

The slopes of both lines are -3.5, so they are parallel.

Therefore, the equation of a line that passes through the point (2, -10) and is parallel to 14x - 2y = 6 is y = -3.5x - 3.

Know more about the slope-intercept form

https://brainly.com/question/1884491

#SPJ11

Other Questions
Find the slope of the tangent to the curve 1/x + 1/y = 1 at the point (2, 2) MRP is the common method to determine the dependent demand. By providing MRP advantages. explain dependent demand. Fast forward to March 2020. The COVID-19 pandemic has hit, and the vast majority of the world is under quarantine. Travel has plummeted, and millions of your customers are canceling their flight reservations. Your fleet sits largely dormant and company debts are piling up quickly.As CEO of American Airlines, your list of concerns is essentially endless right now. You sit down to gather your thoughts on some of the most pressing issues at hand. Address the following legal issues as instructed. why did expansion of the franchise advance the development of political parties? The length of a rectangular plot of land is 5 times the width.If the perimeter is 1000 feet, find the dimensions of the plot.Round to one decimal place if necessary. The next dividend payment by Halestorm, Inc., will be $1.92 per share. The dividends are anticipated to maintain a growth rate of 6 percent forever. If the stock currently sells for $38 per share, what is the required return? Variations in the resistivity of blood can give valuable clues about changes in various properties of the blood. Suppose a medical device attaches two electrodes into a 1.2-mm-diameter vein at positions 5.5cm apart. What is the blood resistivity if a 9.0 V potential difference causes a 280?A current through the blood in the vein? (omega*m) when using an open-source service such as to analyze a file, you should be aware that: 1. One way of screening in the automobile insurance market is for companies toA. offer policies with different deductibles and different premiums.B. eliminate policies with noclaim bonuses because these policies are usually not profitable.C. insure only careful drivers.D. insure only risky drivers.E. offer warranties. a) A merchant receives a shipment of five photocopying machines, two of which are defective. He randomly selects three of the machines and checks them for faults. Let the random variable X be number of faulty machines in his selection. Find the probability distribution of random variable X in the table form. b) Let X be the random variable with the cumulative probability distribution: 0 x < 0 0 x2 F(x) = {0, x Most nations of the world experience are facing an upward pressure on the prices of the majority of goods in the consumer basket. This inflation varies from 5 % to more than 8.5 % on average in the US. Can you list the main causes(variables or factors) of this inflation? Explain why the factors you list are real causes of the US current inflation. 2. How can this inflation affect the gross domestic product in the US? 11. What type of informative speech topic is a speech with the specific purpose:"To inform my audience about the legal definition of privacy"?historybiographyideas or conceptsprocessescategories or divisions12. What type of informative speech topic is a speech with the specific purpose:"To explain to my classmates the life of Mother Teresa"?historybiographyideas or conceptsprocessescategories or divisions13. What type of informative speech topic is a speech with the specific purpose:"To explain to my classmates the four characteristics of a diamond?"historybiographyideas or conceptsprocessescategories or divisions14. A series of speeches by experts in front of an audience with possible question-and-answer afterward is a ____________."forumpanelsymposiumdaisnone of the above What could be taken now to reduce the chances of a similar tragedy occurring in the future? In your answer, please consider what actions, if any, the various individuals and groups mentioned in question 1 could take. Please discuss how the company level, worker level and government level, Suppose you are the house in European Roulette. A bet on asingle number pays 35:1. What is the optimal bet as a percentage ofthe bankroll? while analyzing an intermittent error, james, an independent contractor for hkv infrastructures, finds that the destination host is constantly asking the source to retransmit the data. he finds that the bug might be related to the transport layer of the osi model. since the tcp provides reliable delivery protocols, analyze which of the following characteristics of the tcp protocol james should check to fix this error. Which of the following about superannuation funds is NOT correct? Select one: a. For a defined benefit scheme, the amount of the benefit at maturity depends on the investment performance of the fund. b. It is reasonable for a person nearing retirement to change to a conservative asset allocation for their superannuation fund. c. A defined contribution or accumulation fund involves risk for the member as there is no defined payout. d. A capital guaranteed superannuation fund assures contributors that they will at least receive back the value of their investment. e. The majority of fund members make regular contributions to their superannuation fund over their working life. 4+4 (-3/7) +4 (-3/7)^2+ ...... Find all complex fourth roots of 4. In other words, find all complex solutions of x^4 = 4. Companies buy each other. When this happens, both companies must change to allow the two organizations to become one. But we know, from research, that most mergers do not really succeed. Years later the two companies still haven't integrated How would you take two antagonistic companies perhaps two strong former competitors, or two radically different business types (like merging a Silicon Valley tech company and a 150 year old insurance company) and help the staff in both come together and work together, creating a new common corporate culture. The Empire Inc. is an all-equity firm. It expects perpetual earnings before interest and taxes (EBIT) of $80 million per year. Its equity required return is 12%. The firm is subject to a 25% tax rate and has 10 million shares outstanding.a. What is the value of Empire? What is its share price?b. Empire has an opportunity at an expansion project. The project will require $150 million investment and will generate $40 million pre-tax perpetual annual cash flow. What is Empire Inc. value and share price if it accepts the project and finances it with new equity only?c. Empire can borrow any amount at 10.0%. What is Empire value and share price, if it finances the project with debt? a) Given the psychoacoustic model that signal-to-mask ratios for bands 3, 4, and 5 are for signals above 90 dB in band 4, a masking of 50 dB in band 3, and a masking of 40 dB in band 5. In addition, the signal-to-mask ratios for another three bands 15, 16, 17 are for signals above 100 dB in band 12, a masking of 55 dB in band 11, and a masking of 65 dB in band 13 Six levels of the critical bands of the audio are listed below. Determine which band(s) of data Band 3 Level (dB) 50 4 95 5 20 11 3 12 105 13 70 b) Calculate the number of samples for 3 frames using MPEG-1 Layer 1. c) Continus (b), how many points should be used in the Fast Fourier Transform (FFT)? d) Given the sequence of the Middle/Side channels of a MP3 audio as follows: Side 2 3 -1 0 2 50 0 3 72 Middle 70 12 58 23 3 70 9 45 90 i. Find the sequence of the right channel of the above sequence. Show your work with the aid of equations. ii. Find the sequence of the left channel of the above sequence. Show your work with the aid of equations